2
$\begingroup$

In my professors script for Thermodynamics stated that for different systems there can only be one global affine transformation the reasoning is the following:

The additivity of the entropy of different systems strongly constrains 𝑆(𝑋).
As 𝑆(𝑋) encodes an order, any$$\tilde{𝑆}(𝑋) := 𝛼𝑆(𝑋) +𝛽, 𝛼 > 0$$ is suitable as well for an individual system.
Introducing different 𝛼, 𝛽 for different systems, where$$ 𝑆(𝑋) ≤ 𝑆(𝑋′), 𝑆(𝑌) ≤ 𝑆(𝑌′) \text{ therefore } 𝑆(𝑋)+𝑆(𝑌) ≤ 𝑆(𝑋′)+𝑆(𝑌′)$$would allow for$$\tilde{𝑆}(𝑋) ≤ \tilde{𝑆}(𝑋′), \tilde{𝑆}(𝑌) ≤ \tilde{𝑆}(𝑌'), \text{ but } \tilde{𝑆}(𝑋)+\tilde{𝑆}(𝑌) > \tilde{𝑆}(𝑋′)+\tilde{𝑆}(𝑌′)$$violating eq (6.15) which is:$$ 𝑋 ⪯ 𝑋′ ⇔ 𝑆(𝑋) ≤ 𝑆(𝑋′)$$[find suitable numbers to show this (was ex to the reader)].
Additivity therefore implies the global calibration of entropy across all types of systems:
There can be only one global affine transformation 𝑆(𝑋) → 𝑆˜(𝑋) := 𝛼𝑆(𝑋) + 𝛽, 𝛼 >$0$.
𝛽 will be fixed by the third law of thermodynamics; 𝛼 fixes the unit of entropy.

I am pretty confused because lets say we chose different affine transformations for S(X) and S(Y) such that:
𝑆˜(𝑋) :=$\alpha _1 $S(X) +$\beta _1$ and 𝑆˜(𝑌) :=$\alpha _2 $S(Y) +$\beta _2$ with$\alpha _1 ,\alpha _2 >0$

than:
\begin{align}\tilde S(X)+\tilde S(Y) &> \tilde S(X')+\tilde S(Y') \\[6pt]\Longleftrightarrow\quad\alpha_1 S(X)+\beta_1+\alpha_2 S(Y)+\beta_2&>\alpha_1 \tilde S(X')+\beta_1+\alpha_2 \tilde S(Y')+\beta_2 \\[6pt]\Longleftrightarrow\quad\alpha_1\bigl(S(X)-S(X')\bigr)&>\alpha_2\bigl(S(Y')-S(Y)\bigr)\\[6pt]\end{align}which is contradicted by:$S(X)\le S(X'),\qquad S(Y)\le S(Y'),\qquad \alpha_1,\alpha_2>0.$

askedNov 14 at 11:09
ColBi's user avatar
$\endgroup$
4
  • $\begingroup$Please use Mathjax properly.$\endgroup$CommentedNov 14 at 11:18
  • $\begingroup$Is there a tutorial on that somewhere this is my first question?$\endgroup$CommentedNov 14 at 11:21
  • 2
    $\begingroup$Hi, sorry I forgot to link:MathJax Tutorial.$\endgroup$CommentedNov 14 at 11:24
  • $\begingroup$You are getting the logical inference wrong. You started out with the last-most line, and you want to derive a contradiction. The way to do it is as your professor did it. You, however, went around in a loop. No wonder you get nowhere.$\endgroup$CommentedNov 14 at 18:32

0

Know someone who can answer? Share a link to thisquestion viaemail,Twitter, orFacebook.

Your Answer

Sign up orlog in

Sign up using Google
Sign up using Email and Password

Post as a guest

Required, but never shown

By clicking “Post Your Answer”, you agree to ourterms of service and acknowledge you have read ourprivacy policy.

Start asking to get answers

Find the answer to your question by asking.

Ask question

Explore related questions

See similar questions with these tags.